The Stacks project

10.85 Projective modules over a local ring

In this section we prove a very cute result: a projective module $M$ over a local ring is free (Theorem 10.85.4 below). Note that with the additional assumption that $M$ is finite, this result is Lemma 10.78.5. In general we have:

Lemma 10.85.1. Let $R$ be a ring. Then every projective $R$-module is free if and only if every countably generated projective $R$-module is free.

Proof. Follows immediately from Theorem 10.84.5. $\square$

Here is a criterion for a countably generated module to be free.

Lemma 10.85.2. Let $M$ be a countably generated $R$-module with the following property: if $M = N \oplus N'$ with $N'$ a finite free $R$-module, then any element of $N$ is contained in a free direct summand of $N$. Then $M$ is free.

Proof. Let $x_1, x_2, \ldots $ be a countable set of generators for $M$. We inductively construct finite free direct summands $F_1, F_2, \ldots $ of $M$ such that for all $n$ we have that $F_1 \oplus \ldots \oplus F_ n$ is a direct summand of $M$ which contains $x_1, \ldots , x_ n$. Namely, given $F_1, \ldots , F_ n$ with the desired properties, write

\[ M = F_1 \oplus \ldots \oplus F_ n \oplus N \]

and let $x \in N$ be the image of $x_{n + 1}$. Then we can find a free direct summand $F_{n + 1} \subset N$ containing $x$ by the assumption in the statement of the lemma. Of course we can replace $F_{n + 1}$ by a finite free direct summand of $F_{n + 1}$ and the induction step is complete. Then $M = \bigoplus _{i = 1}^{\infty } F_ i$ is free. $\square$

Lemma 10.85.3. Let $P$ be a projective module over a local ring $R$. Then any element of $P$ is contained in a free direct summand of $P$.

Proof. Since $P$ is projective it is a direct summand of some free $R$-module $F$, say $F = P \oplus Q$. Let $x \in P$ be the element that we wish to show is contained in a free direct summand of $P$. Let $B$ be a basis of $F$ such that the number of basis elements needed in the expression of $x$ is minimal, say $x = \sum _{i=1}^ n a_ i e_ i$ for some $e_ i \in B$ and $a_ i \in R$. Then no $a_ j$ can be expressed as a linear combination of the other $a_ i$; for if $a_ j = \sum _{i \neq j} a_ i b_ i$ for some $b_ i \in R$, then replacing $e_ i$ by $e_ i + b_ ie_ j$ for $i \neq j$ and leaving unchanged the other elements of $B$, we get a new basis for $F$ in terms of which $x$ has a shorter expression.

Let $e_ i = y_ i + z_ i, y_ i \in P, z_ i \in Q$ be the decomposition of $e_ i$ into its $P$- and $Q$-components. Write $y_ i = \sum _{j=1}^{n} b_{ij} e_ j + t_ i$, where $t_ i$ is a linear combination of elements in $B$ other than $e_1, \ldots , e_ n$. To finish the proof it suffices to show that the matrix $(b_{ij})$ is invertible. For then the map $F \to F$ sending $e_ i \mapsto y_ i$ for $i=1, \ldots , n$ and fixing $B \setminus \{ e_1, \ldots , e_ n\} $ is an isomorphism, so that $y_1, \ldots , y_ n$ together with $B \setminus \{ e_1, \ldots , e_ n\} $ form a basis for $F$. Then the submodule $N$ spanned by $y_1, \ldots , y_ n$ is a free submodule of $P$; $N$ is a direct summand of $P$ since $N \subset P$ and both $N$ and $P$ are direct summands of $F$; and $x \in N$ since $x \in P$ implies $x = \sum _{i=1}^ n a_ i e_ i = \sum _{i=1}^ n a_ i y_ i$.

Now we prove that $(b_{ij})$ is invertible. Plugging $y_ i = \sum _{j=1}^{n} b_{ij} e_ j + t_ i$ into $\sum _{i=1}^ n a_ i e_ i = \sum _{i=1}^ n a_ i y_ i$ and equating the coefficients of $e_ j$ gives $a_ j = \sum _{i=1}^ n a_ i b_{ij}$. But as noted above, our choice of $B$ guarantees that no $a_ j$ can be written as a linear combination of the other $a_ i$. Thus $b_{ij}$ is a non-unit for $i \neq j$, and $1-b_{ii}$ is a non-unit—so in particular $b_{ii}$ is a unit—for all $i$. But a matrix over a local ring having units along the diagonal and non-units elsewhere is invertible, as its determinant is a unit. $\square$


Comments (0)


Post a comment

Your email address will not be published. Required fields are marked.

In your comment you can use Markdown and LaTeX style mathematics (enclose it like $\pi$). A preview option is available if you wish to see how it works out (just click on the eye in the toolbar).

Unfortunately JavaScript is disabled in your browser, so the comment preview function will not work.

All contributions are licensed under the GNU Free Documentation License.




In order to prevent bots from posting comments, we would like you to prove that you are human. You can do this by filling in the name of the current tag in the following input field. As a reminder, this is tag 058Z. Beware of the difference between the letter 'O' and the digit '0'.